RowdyHowdy
Editorial: Our province's legislature will soon vote on a measure that would allow government employee unions to bargain over wages. Some critics claim this measure would increase wages and thus exacerbate our province's budget deficit. But in fact, it would probably reduce the deficit. The province immediately north of ours allows such union bargaining, and its budget deficit is lower than our province's—and lower than it was before the province allowed union bargaining.
The editorial's argument is most vulnerable to criticism on which of the following grounds?
A. It fails to adequately address the possibility that many factors affect provincial budget deficits.
B. It confuses a claim about the budget deficit in one province with a similar claim about the deficit in another, neighboring province.
C. It overlooks the possibility that the budget deficit of the province immediately to the north was higher before that province allowed the union bargaining.
D.It takes for granted that if the measure reduces the budget deficit, then it will not increase wages of government employees.
E. It fails to adequately distinguish a factual claim about the measure's likely consequences from a value judgment that the measure should be enacted.
(source: GMAT focus online prep Exam1)
Premises:Some critics claim that union bargaining would increase wages and thus exacerbate our province's budget deficit.
The province immediately north of ours allows such union bargaining, and its budget deficit is lower than our province's—and lower than it was before the province allowed union bargaining.
Conclusion: It would probably reduce the deficit.
The question stem asks us for a flaw in the reasoning. In these questions pre-thinking helps. What is the flaw in the logic here? Because the north province's deficit reduced after allowing union bargaining, the argument is assuming that the same will happen here. The very first issue with that is that we are not given that their deficit reduced because union bargaining reduced expense (and that other things stayed constant). The causality is not given. Their deficit could have reduced because of various other factors. We are not even given what impact union bargaining had on union's budget. It is possible that union bargaining even increased the deficit but other factors led to an overall decrease in the deficit.
A. It fails to adequately address the possibility that many factors affect provincial budget deficits.Correct. It fails to address the possibility that many factors combine to give the budget deficit. Just because two things happened in sequence, it doesn't mean one led to the other.
B. It confuses a claim about the budget deficit in one province with a similar claim about the deficit in another, neighboring province.
The only claim given is "this measure would increase wages and thus exacerbate our province's budget deficit". No such claim about the deficit in another, neighboring province has been mentioned. The author only mentions what actually happened in another, neighboring province.
C. It overlooks the possibility that the budget deficit of the province immediately to the north was higher before that province allowed the union bargaining.
The author actually tells us that the budget deficit of the province immediately to the north was higher before that province allowed the union bargaining. It is not something he has overlooked
D.It takes for granted that if the measure reduces the budget deficit, then it will not increase wages of government employees.
The author does not talk about wages at all. He doesn't say whether the wages will increase or decrease etc.
E. It fails to adequately distinguish a factual claim about the measure's likely consequences from a value judgment that the measure should be enacted.
We do not know whether the claim given is "factual claim" (can be proven to be true or false). Hence we cannot say that the author's argument is flawed on this account. This is not the flaw that we can identify in this argument.
Answer (A)